Write down inequalities,that are satisfied by these sets of integers between -10 and 10
1,2,3,4,5,6,7,8,9,10
-3,-4,-5,-6,-7,-8,-9,-10
9,10
-10

Answers

Answer 1

Answer:

Below

Step-by-step explanation:

Notice that x is between 10 and -10 but takes only the values that are integers.

The inequalities:

● we can write an inequality that includes all these values.

● -10 《 x 《 10

This is a possible inequality

Multiply both sides by 2 and you will get a new one:

● -20 《 2x 《 20

You can multiply it by any number to generate a new inequality.

Or you can add or substract any number.


Related Questions

What is the value of z for the equation fraction 1 over 2z = −fraction 3 over 4 + fraction 1 over 4z? −3 −1 1 3

Answers

Answer:

z= -3

Step-by-step explanation:

Given:

1/2z =-3/4 + 1/4z

Collect like terms

1/2z - 1/4z = -3/4

Add 1/2z - 1/4z

2z-z / 4 = -3/4

We have

z/4=-3/4

Same as

z(1/4) = -3/4

Divide both sides by 1/4

z(1/4) ÷ 1/4 = -3/4÷1/4

z(1/4) × 4/1= -3/4 × 4/1

z(4/4) = -12/4

z= -3

The value of z= -3

Answer:

-3

Step-by-step explanation:

I got it right on the test

Marta esta poniendo sus libros en una estantería. Le faltan 7 libros para poder poner 12 en cada estante; sin embargo, si pone 10 libros en cada estante, se quedan 5 libros sin poner. ¿Cuantos es antes tiene la estantería?

Answers

Answer:

x = 6       la cantidad de estantes

y = 65    cantidad de libros

Step-by-step explanation:

LLamemos "x" la cantidad de estantes que tiene Marta, y llamaremos "y" la cantidad de libros.

La primera condición que se debe cumplir es que cuando Marta coloca 12 libros en cada estante entonces le faltan 7, esto lo expresamos así:

y  +  7  = 12*x        (1)

La segunda condición establece que si Marta coloca los libros en número de 10 por estante le quedan 5 sin colocar, luego esto en lenguaje matemático se expresa así:

y  -   5   = 10*x     (2)

Ahora hemos obtenido un sistema de dos ecuaciones con dos incógnitas que se resuelve por cualquiera de los métodos conocidos, usaremos el método de sustitución.

Despejamos   y en la primera ecuación y lo sustituimos en la segunda, de esa forma obtendremos el valor de x

y  =  12*x  - 7

(12*x - 7 ) - 5  = 10*x

2*x  -12 = 0

2*x = 12

x = 6       la cantidad de estantes, y

y  =  12*x -7

y  =  72  -  7

y =  65    cantidad de libros

Consider f(x) = 3x2 + 4 and g(x) = 2x − 3. Which statements about f + g are true? Select all that apply. A. The domain is all real numbers. B. The range is all real numbers. C. It is a linear function. D. It is a quadratic function.

PLEASE HELP

Answers

Answer:

B. The range is all real numbers. D. It is a quadratic function.

Step-by-step explanation:

Given the functions  f(x) =  3x² + 4 and g(x) = 2x − 3, to know that statements that is  true about f+g, we will need to add the functions together first.

f(x)+g(x) = 3x² + 4 + 2x - 3

f(x)+g(x) = 3x²+2x +4 - 3

f(x)+g(x)  = 3x²+2x + 1

The highest degree of a quadratic function is 2 and since the highest degree of the resulting function is 2, hence the resulting sum of the equations is quadratic.

Also the range of the values is all real numbers because the presence of x² in the function will always return a positive value greater than x.

Hence the correct statements are 'the range is all real numbers and it is a quadratic function'

Answer:

B AND D

Step-by-step explanation:

Allison is rolling her hula hoop on the playground. The radius of her hula hoop is 35 \text{ cm}35 cm35, start text, space, c, m, end text. What is the distance the hula hoop rolls in 444 full rotations?

Answers

Answer: 880 cm

Step-by-step explanation:

Given: Radius of the hula hoop = 35 cm

Hula hoop is  circular in shape

Then, Circumference = [tex]2\pi r[/tex] , where r = radius

Now , Circumference of hula hoop = [tex]2\times \dfrac{22}{7}\times35=220\ cm[/tex]

Now , the distance the hula hoop rolls in 4 full rotations = 4 × (Circumference of hula hoop)

[tex]= 4 \times 220=880\ cm[/tex]

Hence, the required distance = 880 cm

Answer:

880

Step-by-step explanation:

Please help!! Thank you in advance! Will mark Brainliest!

Answers

Answer:

F , D, E

Step-by-step explanation:

The smallest angle is opposite the smallest side

The largest angle is opposite the largest side

DE is smallest so F is smallest

Then EF so D

DF is largest so E is the largest angle

F, D, E

Step by Step

solve for x 5(x+1)=4(x+8)

Answers

Answer:

x=27

Step-by-step explanation:

expanding the above expression we get

5x+5=4x+32

grouping numbers with coefficient of x at the left side and constant at the right side we get

5x-4x=32-5

x=27

To solve this equation, we start by distributing both the 5 and the 4 through both set of parentheses.

This gives us 5x + 5 = 4x + 32.

Now subtract 4x from both
sides to get x + 5 = 32.

Now subtract 5 from both sides and x = 27.

There are a total of two hundred students and chaperones going on a
field trip. Each bus can hold 60 passengers. How many buses will be
used for the field trip? Explain why your answer is reasonable.

Answers

Answer:

4 bus is required for field trip to carry 200 passengers.

Step-by-step explanation:

Total no . of passengers = 200

let the be x bus required to carry 200 passengers

capacity of 1 bus = 60 passengers

capacity of x bus = 60*x passengers = 60x passengers

Thus,

60x = 200

x = 200/60 = 3 2/3

thus, 3.66 bus is required , but no. of bus cannot be in fraction hence we take integral value greater than 3.66 which is 4

Thus, 4 bus is required for field trip to carry 200 passenger.

Evaluate -3.28 - (-4.4) + (-p) where p = 9.7

Answers

Answer:

-8.58

Step-by-step explanation:

p = 9.7

=> -3.28 - (-4.4) + (-p)

=> -3.28 - (-4.4) + (- (9.7))

=> -3.28 - (-4.4) + (-9.7)

=> -3.28 - (-4.4) -9.7

=> -3.28 +4.4 -9.7

=> -12.98 +4.4

=> -8.58

Answer:

The evaluated answer for this equation is -8.58

Step-by-step explanation:

We are given a value of p so we will use this value for the equation.

-3.28 - (-4.4) + (-p)

Replace p with 9.7

-3.28 - (-4.4) + (-9.7)

Since there is a double negative in front of 4.4, then that means that we are going to be adding -3.28 to 4.4

1.12 + (-9.7)

Subtract 1.42 from 9.7 because there is negative sign in front of 9.7

-8.58

What are the domain and range of the real-valued function f(x)=2/(x+5)?

Answers

Answer:

Domain is all real numbers, x ≠ -5

Range is all real numbers, y ≠ 0

Step-by-step explanation:

Domain: all real numbers except x=-5
(-infinity, -5)U(-5, infinity)

Range: all real numbers except y=0
(-infinity, 0)U(0, infinity)

HELP!!!
The solutions to (x + 3)^2- 4=0 are x = -1 and x = -5

True or false

Answers

Answer:

False

Step-by-step explanation:

We can simplify this equation and then solve for x.

[tex](x+3)^3-4=0\\\\x^2+6x+9-4=0\\\\x^2+6x+5=0\\\\(x+2)(x+3)=0\\\\x=-3\\x=-2[/tex]

As you can see, the solutions are not x=-1 and x=-5.

Therefore, the answer is false.

Answer:

True

Step-by-step explanation:

Given

(x + 3)² - 4 = 0 ( add 4 to both sides )

(x + 3)² = 4 ( take the square root of both sides )

x + 3 = ± [tex]\sqrt{4}[/tex] = ± 2 ( subtract 3 from both sides )

x = - 3 ± 2

Thus

x = - 3 - 2 = - 5

x = - 3 + 2 = - 1

If you invest $600 at 5% interest compounded continuously, how much would you make after 6 years?

Answers

Answer:

809.915$

Step-by-step explanation:

Amount of money = Principal x e^(rate x year)

                              = 600 x e^(0.05 x 6)

                              = 809.915$

Answer:

$809.92

Step-by-step explanation:

(see attached for reference)

Recall that the formula for compound interest (compounded continuously) is

A = P e^(rt)

where,

A = final amount (we are asked to find this)

P = principal = given as $600

r = interest rate = 5% = 0.05

t = time = 6 years

e = 2.71828 (mathematical constant)

Substituting the known values into the equation:

A = P e^(rt)

= 600 e^(0.05 x 6)

= 600 (2.71828)^(0.30)

= $809.92

A car is averaging 50 miles per hour. If the car maintains this speed, how many minutes less would a 450-mile trip take than a 475-mile trip?

Answers

Answer:

1/2 a minute (30 seconds)

Step-by-step explanation:

475/50=9.5

450/50=9

9-9.5=.5

**Yoxelt buys 4 1/ 2 gallons of soda. One-fourth of the soda he bought was Pepsi and the rest was Sprite. How many gallons of Pepsi did Yoxelt buy? Show all work below.

Answers

Answer:

1 1/8

Step-by-step explanation:

1/4 of 4 1/2 is Pepsi.

1/4 * 4 1/2 = (1/4) * 4 + (1/4) * (1/2) = 1 1/8

help..? why are there so many parentheses..?can you plz give a step by step on how to slove the equation?

Answers

Answer:

= -11

Step-by-step explanation:

-(-(11-22))

= -(-11+22)

= 11 - 22

= -11

Solve logs (8 - 3x) = log20 for x.
A. X = 14
B. X = -13
C.x = -8
D. X= -4

Answers

Answer:

x = -4

Step-by-step explanation:

logs (8 - 3x) = log20

Since we are taking the log on each side

log a = log b  then a = b

8 -3x = 20

Subtract 8  from each side

8 -3x-8 =20 -8

-3x = 12

Divide by -3

-3x/-3 = 12/-3

x = -4

Answer:

[tex] \boxed{\sf x = -4} [/tex]

Step-by-step explanation:

[tex] \sf Solve \: for \: x \: over \: the \: r eal \: numbers:[/tex]

[tex] \sf \implies log(8 - 3x) = log 20[/tex]

[tex] \sf Cancel \: logarithms \: by \: taking \: exp \: of \: both \: sides:[/tex]

[tex] \sf \implies 8 - 3x = 20[/tex]

[tex] \sf Subtract \: 8 \: from \: both \: sides:[/tex]

[tex] \sf \implies 8 - 3x - 8 = 20 - 8 [/tex]

[tex] \sf \implies - 3x = 12 [/tex]

[tex] \sf Divide \: both \: sides \: by \: - 3:[/tex]

[tex] \sf \implies \frac{-3x}{-3} = \frac{12}{-3} [/tex]

[tex] \sf \implies x = - 4[/tex]

what is the radical of 5√72 PLZ HELP!

Answers

Answer: Exact Form: 30√2

Decimal Form:42.42640687…

Step-by-step explanation: Simplify the radical by breaking the radicand up into a product of known factors, assuming positive real numbers.

I hope this helped :)

Answer:

  30√2

Step-by-step explanation:

The radical portion of the given expression is √72.

__

Perhaps you want the simplest form of your expression. Factor out the perfect squares from under the radical.

  [tex]5\sqrt{72}=5\sqrt{36}\sqrt{2}=5\cdot 6\sqrt{2}=\boxed{30\sqrt{2}}[/tex]

What property is demonstrated here? (3x-5) x 4 = 3 x (-5 x 4) A) commutative property of addition B) associative property of multiplication C) commutative property of multiplication D) associative property of addition (haven't learned this yet so I have no clue)

Answers

Answer:

B) Associative Property of Multiplication

Step-by-step explanation:

*if it's wrong idk how, but I apologise*

The row-echelon form of the augmented matrix of a system of equations is given.Find the solution of the system

Answers

Answer:

x = 9/4

y = 3/5

z = 2/3

w = -9/5

Step-by-step explanation:

Technically, the matrix is in reduced row echelon form. If there are zeros above and below the ones, it is RREF. If there are zeros only below the ones, then it's REF.

Since it is in RREF, the augmented numbers to the right of the bar are already your solutions. Simply label the variables.

asap!!
~~~~~~
A line passes through point (–6, –1) and is parallel to the equation y = –2x – 5. What's the equation of the line?

Question 25 options:

y = –2x – 13

y = 12{"version":"1.1","math":"\(\frac{1}{2}\)"}x + 3

y = –12{"version":"1.1","math":"\(\frac{1}{2}\)"}x – 1

y = 2x + 5

click on picture for a, b, c ,or d

Answers

Answer:

y=−2x−13.

Step-by-step explanation:

The equation of the line in the slope-intercept form is y=−2x−5.

The slope of the parallel line is the same: m=−2.

So, the equation of the parallel line is y=−2x+a.

To find a, we use the fact that the line should pass through the given point: −1=(−2)⋅(−6)+a.

Thus, a=−13.

Therefore, the equation of the line is y=−2x−13.

What is another way to write 100,203 in other forms

Answers

Answer:

You can write in

Step-by-step explanation:

Lakhs

First write 100,203.

Put ones,tens,hundreds,thousands,ten thousands and lakh on top of each number from the extreme left.

This is how you can write 100,203 in another way. You can't write in any other way than this one.

Hope this helps....

Have a nice day!!!!

state the vertical distance and horizontal distance of the two pairs of points given.​

Answers

Answer:

the horizontal distance is the x-intercept, and the vertical distance is the y-intercept

1) x-int=1, y-int=1

2) x-int=6, y-int=5

Using a table of values, determine the solution to the equation below to the nearest fourth of a unit. 2^x=1-3^x

Answers

Answer:

Option (1)

Step-by-step explanation:

Given equation is,

[tex]2^x=1-3^x[/tex]

To determine the solution of the equation we will substitute the values of 'x' given in the options,

Option (1)

For x = -0.75

[tex]2^{-0.75}=1-3^{-0.75}[/tex]

0.59 = 1 - 0.44

0.59 = 0.56

Since, values on both the sides are approximately same.

Therefore, x = -0.75 will be the answer.

Option (2)

For x = -1.25

[tex]2^{-1.25}=1-3^{-1.25}[/tex]

0.42 = 1 - 0.25

0.42 = 0.75

Which is not true.

Therefore, x = -1.25 is not the answer.

Option (3)

For x = 0.75

[tex]2^{0.75}=1-3^{0.75}[/tex]

1.68 = 1 - 2.28

1.68 = -1.28

Which is not true.

Therefore, x = 0.75 is not the answer.

Option (4)

For x = 1.25

[tex]2^{1.25}=1-3^{1.25}[/tex]

2.38 = 1 - 3.95

2.38 = -2.95

It's not true.

Therefore, x = 1.25 is not the answer.

BRAINLIEST, 5 STARS AND THANKS IF ANSWERED CORRECTLY.

A quadratic equation with a negative discriminant has a graph that..

A. touches the x-axis but does not cross it
B. opens downward and crosses the x-axis twice
C. crosses the x-axis twice.
D. never crosses the x-axis.

Answers

Answer:

never crosses the x-axis.

Step-by-step explanation:

     A quadratic equation with a negative discriminant has a graph that - never crosses the x-axis.

Answer:

The graph of a quadratic equation that has a negative discriminant is the one that never intersect x-axis. The graph of a quadratic equation that has a zero discriminant is the one that intersect x-axis at only one point. To be clearer, it can be seen in the attached image.

Step-by-step explanation:

Answer D

PLEASE HELPP on THIS PICTURE FOR ONE OF MY QUESTIONS

Answers

Answer:

Linear pair postulate

Step-by-step explanation:

The Linear Pair Postulate states: "If two angles form a linear pair, then the angles are supplementary; that is, the sum of their measures is 180 degrees

A linear pair of angles is such that the sum of angles is 180 degrees.

Barry has been watching the geese that live in his neighborhood. The number of geese changes each week. n f(n) 1 56 2 28 3 14 4 7 Which function best shows the relationship between n and f(n)? f(n) = 28(0.5)n f(n) = 56(0.5)n−1 f(n) = 56(0.5)n f(n) = 112(0.5)n−1

Answers

Answer:

B. f(n) = 56(0.5)^n-1

Step-by-step explanation:

 First, You have to find out the starting population, if you look at the problem you see the population starts at 56

f(x) = 56

Second, you know that the population goes down 50% each week so it has a decay of 0.5

f(x) = 56(0.5)

 Third,  you need to add the exponent of n to make it exponential.  But, if you just add n then the the population would be 28 on week 1 which is incorrect. To fix that you make the exponent n-1 so when you are on week 1 it doesn't become 28 but it stays on 56, and on week 2 it's 28, ect

f(x) = 56(0.5)^n-1

Simplify the expression. (3x2 – 4x + 1) + (-x2 + x – 9)

Answers

[tex](3x^2 - 4x + 1) + (-x^2 + x - 9)=\\3x^2-4x+1-x^2+x-9=\\2x^2-3x-8[/tex]

= (6x+4x+1)+(-2x+x-9)
= (2x+1)+(-x-9)
= 2x+1-x-9
= x-8

Which of the following lists of three numbers could form the side lengths of a triangle? A. 10, 20, 30 B. 122, 257, 137 C. 8.6, 12.2, 2.7 D. 1/2, 1/5, 1/6

Answers

Answer:

Step-by-step explanation:

The triangle inequality theorem states that the sum of any two sides of a triangle os greater than the third side.

■■■■■■■■■■■■■■■■■■■■■■■■■■

First triangle:

Let a,b and c be the sides of the triangle:

● a = 10

● b = 20

● c = 30

Now let's apply the theorem.

● a+b = 10+20=30

That's equal to the third side (c=30)

●b+c = 50

That's greater than a.

● a+c = 40

That's greater than b.

These aren't the sides of a triangel since the first inequality isn't verified.

■■■■■■■■■■■■■■■■■■■■■■■■■

Second triangle:

● a = 122

● b = 257

● c = 137

Let's apply the theorem.

● a+b = 379

That's greater than c

● a+c = 259

That's greater than b

● b+c = 394

That's greater than a

So 122,257 and 137 can be sides of a triangle.

■■■■■■■■■■■■■■■■■■■■■■■■■■

The third triangle:

● a = 8.6

● b = 12.2

● c = 2.7

Let's apply the theorem:

● a+b = 20.8

That's greater than c

● b+c = 14.9

That's greater than a

● a+c = 11.3

That isn't greater than b

So theses sides aren't the sides of triangle.

■■■■■■■■■■■■■■■■■■■■■■■■■■

● a = 1/2

● b = 1/5

● c = 1/6

Let's apply the theorem.

● a+b = 7/10

That's greater than c

● a+c = 2/3

That's greater than b

● b+c = 11/30

That isn't greater than a

So these can't be the sides of a triangle.

Given the right triangle below, if AB = 4 and BC = 4, find AC.
A
B
C

Answers

The length of AC is found using the Pythagorean theorem which is a^2+b^2=c^2.

In this case your equation would look like 4^2+4^2=c^2.
Four to the power of two is 16 so 16+16=c^2.
16+16=32
32=c^2
Root 32 = c

Length AC is root 32.

AC will be 4√2 when AB = 4 and BC = 4, in the given right triangle.

What is Pythagoras' Theorem?

According to Pythagoras' Theorem, in a right triangle, the square of the length of the longest side, that is, the hypotenuse, that is, the side opposite to the right angle is equal to the sum of the squares of the lengths of the other two sides.

How to solve the question?

In the question, we are given a right triangle, with sides AB = 4 and BC = 4.

We are asked to find AC.

To find AC, we will use the Pythagoras theorem, according to which, we can write:

AC² = AB² + BC²

or, AC² = 4² + 4²,

or, AC² = 16 + 16,

or, AC² = 32,

or, AC = √32,

or, AC = √(16 * 2) = 4√2.

Therefore, AC will be 4√2 when AB = 4 and BC = 4, in the given right triangle.

Learn more about Pythagoras' Theorem at

https://brainly.com/question/231802

#SPJ2

1A-MATH
Diviwuetalls
2 Exercise 7:
Your ansv
Type you
The three angles in a triangle are labeled A, B and C.
A = 2x
B= 3x
C=4x
Private coi
1) calculate X
2) find the actual angle of B
3) find the exterior angle to B

Answers

Answer:

Value of x = 20°

Angle B = 60°

Exterior angle to B = 300°

Step-by-step explanation:

Given:

Angles of triangle.

A = 2x

B = 3x

C = 4x

We know that,

A + B + C = 180°

2x + 3x + 4x = 180°

9x = 180°

x = 20°

Value of x = 20°

Angle B = 3x

Angle B = 3(20°)

Angle B = 60°

Exterior angle to B = 360° - 60°

Exterior angle to B = 300°

Which expression is equivalent to (–2)(a + 6)?
A. –2a + 6
B. 2a + 12
C. –2a – 12
D. –2a + 12

Answers

The answer is option c.

Other Questions
Which uses of soil are discussed in the video? Check all that apply. as a habitat for animals to live in as a way to clean and store water as a place for plants and crops to grow as a material to clean and smooth skin as a building material for homes and buildings as an area to build windmills for energy production The main difference between archaeologists and anthropologists is thatarchaeologists find and excavate the artifacts of ancient people, while anthropologists excavate the tombs of ancient people.archaeologists concentrate on the physical artifacts of ancient people, while anthropologists study a wider range of human cultures and behaviors.archaeologists search for and excavate the artifacts of ancient people, while anthropologists interpret the findings of archaeologists.archaeologists focus on artifacts of ancient people, while anthropologists study human remains of ancient people. why would it be essential for the successful A/V technician to participate in additional coursework, presentations, and seminars offered by equipment manufacturers as well as annual conferences attended by colleagues in the industry A ____ is marked by two sets of double yellow lines, with each set having a broken line on the inside, and a solid line on the outside. white arrows appear in this lane as well. The average monthly rainfall for 6 months was 28.5 mm. If it had rained 1mm more each month what would the average have been? By how much would the total have been increased in six months and by how much would average have been increased per month? What's the answer?? Which of the following is not a function of Mitosis? A. Growth and development B. Produce genetically identical daughter cells C. Produce genetically different daughter cells D. Replace damaged cells A professor of women's studies is interested in determining if stress affects the menstrual cycle. Ten women are randomly sampled for an experiment and randomly divided into two groups. One of the groups is subjected to high stress for two months while the other lives in a relatively stress-free environment. The professor measures the menstrual cycle (in days) of each woman during the second month. The following data are obtained. High stress 20 23 18 19 22Relatively stress free 26 31 25 26 30Required:1. The obtained value of the appropriate statistic is _______a. tobt = -4.73b. tobt = -4.71c. tobt = -3.05d. tobt = -.0472. The df for determining tcrit are ____.a. 4b. 9c. 8d. 33. Using a = 0.052 tail, tcrit = ____.a. +- 2.162b. +- 2.506c. +- 2.462d. +- 2.3064. Using a = .052 tail, your conclusion is ____.a. Accept H0; stress does not affect the menstrual cycleb. Retain H0; we cannot conclude that stress affects the menstrual cyclec. Retain H0; Stress affects the menstrual cycled. Reject H0; stress affects the menstrual cycle In the adjoining figure, in ABC, D is the midpoint of the side BC.Hence a) AD is ___________ A b) AM is ____________ c) Is BD = DC please help!!!!!!!!!!!!!! what is the right form for mla format? For each bond, show the direction of polarity by selecting the correct partial charges. _________ Si-P _________ _________ Si-Cl _________ _________ Cl-P _________ The most polar bond is _______ (4) The Highest Common Factor of 35pq^2, -14pq^2 and -21p^2q^3 is We have two fractions, \dfrac{1}{6} 6 1 start fraction, 1, divided by, 6, end fraction and \dfrac{3}{8} 8 3 start fraction, 3, divided by, 8, end fraction, and we want to rewrite them so that they have a common denominator (and whole number numerators). What numbers could we use for the denominator? Choose 2 answers: Choose 2 answers: (Choice A) A 121212 (Choice B) B 242424 (Choice C) C 161616 (Choice D) D 4848 What does the underlined word mean in the following sentence? Me gusta mucho manejar. to travel to rest to drive to turn Is MNO=PQR? If so, name the congruence postulate that applies.Given:ME=PQNO=QRMO=PRA. Congruent - ASAB. Congruent - SSSC. Congruent - SASD. Might not be congruent Your mother calls you and asks you to help with a major family decision. Your maternal grandfather is 70 years old and has been diagnosed with a condition that will kill him some time in the next five years. He can have a procedure that will correct the disease and not leave him with any long-term problems, but the procedure has a 10% mortality rate. He wants to have the procedure, but your mother does not want him to. How would you help mediate this issue? aap kis vyakti se prabhavit rahe hain aur kyon Write a brief description about your visit to the Niagara Falls in not more than 120 words.( mention location, atmosphere, sights, sounds, history, importance etc.) Knowing she has sold 5,000 pairs, assume the company wants to launch a Black Friday promotion, where she would discount her shoes by 10%. How many more shoes would she have to sell to justify this promotion The new hybrid car can get 51.5 km/gal. It has a top speed of 40000.00 cm/min and is 4m long. How fast can the car go in m/hr?